LSAT and Law School Admissions Forum

Get expert LSAT preparation and law school admissions advice from PowerScore Test Preparation.

 voodoochild
  • Posts: 185
  • Joined: Apr 25, 2012
|
#6206
Why is A correct? The premise states that banks contributed to the decline by loaning less money. I am not sure why we are doubting the above statement.

I chose B. I didn't like any other answer choice. The argument is assuming that Tight RS---cause---> less money.

The complete chain of thought is: [Tight RS ---cause--->]less money :arrow: economic decline.

(dotted arrow and [] = assumption; arrow = premise)


Please help :(

Thanks
Voodoo Child
 Nikki Siclunov
PowerScore Staff
  • PowerScore Staff
  • Posts: 1362
  • Joined: Aug 02, 2011
|
#6208
Well, nobody is doubting the statement that banks contributed to the decline by loaning less money. However, the author attributes the bank's reluctance to loan money to tightened regulatory standards, arguing that banks will lend more money if those standards are relaxed. The author makes the following causal argument:

Tightened regulation (cause) :arrow: Banks loan less money (effect)

As is usually the case with causal arguments followed by an Assumption question, we are looking for a Defender Assumption stating that an alternative cause does not exist. (A) fits the bill, claiming that the downturn did NOT cause a decrease in the amount of money available for lending. If, however, the downturn did cause such a decrease, that would be a valid reason why banks loaned less money: they just didn't have as much money to begin with:

Total money decreased (cause) :arrow: Banks loan less money (effect)

The regulatory standards would have no bearing on this causal relationship, which is why the negation of answer choice (A) weakens the argument: it gives an alternative cause to the effect.
 voodoochild
  • Posts: 185
  • Joined: Apr 25, 2012
|
#6214
Nikki,
Thanks for your reply.

I agree with this
Nikki Siclunov wrote: Tightened regulation (cause) :arrow: Banks loan less money (effect)
I am still unclear. Let me dissect this argument:

Premise : banks' less money => economic decline.

AS you said, the conclusion is about : tight standards caused less money for banks.

Nikki Siclunov wrote: (A) fits the bill, claiming that the downturn did NOT cause a decrease in the amount of money available for lending.
However, we know from premise that banks' less money =caused=> decline. Why are we trying to defend this causality by stating that downturn didn't cause the decline in money available for lending? A) doesn't talk anything about the implicit causality between tight reg. standard =cause=> less money in the conclusion--- this causality is the main point of the argument.


I am still not sure.

Please help me :(
 Nicholas Bruno
PowerScore Staff
  • PowerScore Staff
  • Posts: 62
  • Joined: Sep 27, 2011
|
#6217
You are right about the main point/conclusion in this argument.

However, remember there are two types of assumption answers: 1) "Supporter" assumption - where the assumption is the "missing link" in the argument and 2) "Defender" assumptions - where you are trying to identify the answer choice that would eliminate a possible argument against the author's argument in the stimulus.

So going back to the stimulus: the statement we are focusing on is the statement that tightened standards cause less money for the banks. Answer choice A eliminates an alternative cause (namely, that economic downturn (NOT the tightened lending standards) caused the banks to have less money to lend).

Basically, don't think about this answer choice in terms of the "supporter" assumption question - this answer choice "defends" the answer choice from an alternative cause.

I hope that helps!
 voodoochild
  • Posts: 185
  • Joined: Apr 25, 2012
|
#6219
Nicholas,

Thanks for your reply. I agree with you 'supporter'/'defender' explanation. I believe that this example is tricky because it deals with two causal connections - one in the premise and the other in the conclusion.


Tight restrictions ===CAUSE====> less money ----CAUSE--------> economic decline

"====" : Conclusion
"-----" = Premise

A actually defends by saying that economic decline didn't cause less money. One way we can argue that because we already know the fact that less money =causes=> economic decline, why are we doubting it? Or why are we saying that economic decline ==didnot cause====> less money. On the other hand, one could argue that because the conclusion explains what caused "less money with banks", economic decline could NOT have led to less money.

there are two schools of thought. I guess since LSAT thinks A) is correct, we have go with it.

Do you think that my understanding is correct? Or I am missing something.

Please let me know

thanks
 Nikki Siclunov
PowerScore Staff
  • PowerScore Staff
  • Posts: 1362
  • Joined: Aug 02, 2011
|
#6228
I think you are confusing two things: banks having less money with banks loaning less money. The author never stated that banks having less money caused the downturn. The point was that they loaned less money due to the tight regulatory standards.

Here's the causal chain in a nutshell:

Tight regulatory standards :arrow: banks loaned less money :arrow: downturn

If (A) weren't true, i.e. if the downturn caused a decrease in the total amount of money available for lending, then it is no longer the regulatory standards that can be faulted for restricting the money supply available for lending.
 voodoochild
  • Posts: 185
  • Joined: Apr 25, 2012
|
#6261
Nikki,

Thanks for your reply. In my reply, when I said "less money", I implied 'loan less money.' However, I think that my question is a bit different.
Nikki Siclunov wrote: Here's the causal chain in a nutshell:

Tight regulatory standards :arrow: banks loaned less money :arrow: downturn

If (A) weren't true, i.e. if the downturn caused a decrease in the total amount of money available for lending, then it is no longer the regulatory standards that can be faulted for restricting the money supply available for lending.
However, the premise clearly states that bank loaned less money :arrow: downturn. How could A) be true? (A says that downturn didn't play a role in banks inability to loan money. As a matter of fact, we already know this! The first statement of the argument clearly says this)

Can you please help me?

Thanks
 GGTR
  • Posts: 2
  • Joined: Oct 25, 2012
|
#6271
voodoochild wrote:
Can you please help me?
Haven't they kind of helped you repeatedly? You're making the issue more confusing with your questions, which miss the point being made.
 voodoochild
  • Posts: 185
  • Joined: Apr 25, 2012
|
#6273
Mr. GGTR,
I am surprised that you joined yesterday, and that you have posted only one message. I am really disappointed by seeing your -ve post. If I were in your place, I would have helped the poster. We are here to help each other, and not to demean someone. I am sorry for being rude.

Well, let me answer your question. I tried solving this question about 4-5 times. I missed it every time I solved it. I had to spend quite a few hours to analyze why I am missing this question. As you would have noticed, I am looking for a specific explanation for WHY A) is correct.

After reading my posts, you would have noticed that my understanding of the argument is 100% accurate. However, the issue is that answer choice A) talks about a causal chain which is already given in the premise (First statement of the argument). If the premise says that X =causes=> y ; it is known that y doesn't cause x. How could that be an assumption?

Can someone please help me?


Thanks
 GGTR
  • Posts: 2
  • Joined: Oct 25, 2012
|
#6275
I've lurked here for ages, so what does it matter when I joined? And I'm a girl, not a guy, thanks for assuming otherwise.

My point was simple: you plaintively (look it up) ask for help after they gave you 3 good answers. And did you read what they wrote? I thought they explained it very clearly. That's all I was sayin'--don't take it so personally!

"Mr." GGTR

Get the most out of your LSAT Prep Plus subscription.

Analyze and track your performance with our Testing and Analytics Package.